Nephrology Quiz

Description

20 cases in 30 minutes
Mostapha Abdelkader
Quiz by Mostapha Abdelkader, updated more than 1 year ago
Mostapha Abdelkader
Created by Mostapha Abdelkader almost 8 years ago
332
1

Resource summary

Question 1

Question
Q.R.S. is a 60-year-old (72 kg) patient in the intensive care unit. He suffered a myocardial infarction about 1 week ago with secondary heart failure. He now has pneumonia. He has been hypotensive for the past 5 days. Before his admission 1 week ago, he had an SCr of 1.0 mg/dL. His medical history is significant for diabetes mellitus and hypertension. His urine output has been steadily declining for the past 3 days, despite adequate hydration. He made only 700 mL of urine during the past 24 hours. His medications since surgery include intravenous dobutamine, nitroglycerin, and cefazolin. Yesterday, his BUN and SCr were 32 and 3.1 mg/dL, respectively; today, they are 41 and 3.9 mg/dL. His urine osmolality is 290 mOsm/kg. His urine Na is 40 mEq/L, and there are tubular cellular casts in his urine. Which one of the following is the most likely renal diagnosis?
Answer
  • A. Prerenal azotemia.
  • B. ATN.
  • C. Acute interstitial nephritis (AIN).
  • D. Hemodynamic/functional-mediated acute kidney injury (AKI).

Question 2

Question
R.T. is a 60-year-old HD patient who has had ESRD for 10 years. His HD access is a left arteriovenous fistula. He has a history of hypertension, CAD, mild CHF, type 2 diabetes mellitus, and a seizure disorder. Medications: Epoetin 14,000 units 3 times/ week at dialysis; multivitamin (Nephrocaps) once daily; atorvastatin 20 mg/day; insulin; calcium acetate 2 tablets 3 times/day with meals; phenytoin 300 mg/day; and intravenous iron 100 mg/month. Laboratory values: Hemoglobin 10.2 g/dL; immunoassay for PTH (iPTH) 800 pcg/mL; Na 140 mEq/L; K 4.9 mEq/L; Cr 7.0 mg/dL; calcium 9 mg/dL; albumin 2.5 g/dL; and phosphorus 7.8 mg/dL. Serum ferritin is 200 ng/mL, and transferrin saturation is 32%. The RBC indices are normal. His WBC is normal. He is afebrile. 1)- Which one of the following is most likely contributing to relative epoetin resistance in this patient?
Answer
  • A. Iron deficiency.
  • B. Hyperparathyroidism.
  • C. Phenytoin therapy.
  • D. Infection

Question 3

Question
R.T. is a 60-year-old HD patient who has had ESRD for 10 years. His HD access is a left arteriovenous fistula. He has a history of hypertension, CAD, mild CHF, type 2 diabetes mellitus, and a seizure disorder. Medications: Epoetin 14,000 units 3 times/ week at dialysis; multivitamin (Nephrocaps) once daily; atorvastatin 20 mg/day; insulin; calcium acetate 2 tablets 3 times/day with meals; phenytoin 300 mg/day; and intravenous iron 100 mg/month. Laboratory values: Hemoglobin 10.2 g/dL; immunoassay for PTH (iPTH) 800 pcg/mL; Na 140 mEq/L; K 4.9 mEq/L; Cr 7.0 mg/dL; calcium 9 mg/dL; albumin 2.5 g/dL; and phosphorus 7.8 mg/dL. Serum ferritin is 200 ng/mL, and transferrin saturation is 32%. The RBC indices are normal. His WBC is normal. He is afebrile. 2)- In addition to diet modification and emphasizing adherence, which one of the following is the best approach to managing this patient's hyperparathyroidism and renal osteodystrophy?
Answer
  • A. Increase calcium acetate
  • B. Change calcium acetate to sevelamer and add cinacalcet.
  • C. Hold calcium acetate and add intravenous vitamin D analog.
  • D. Add intravenous vitamin D analog.

Question 4

Question
R.Z. is a 76-year-old woman with an acute febrile illness that includes some diarrhea and generalized aches. She has been taking ibuprofen for pain for the past 48 hours and presents to the emergency department feeling “awful.” Her laboratory tests and physical examination suggest she is not volume depleted. Her SCr has doubled since her last visit 1 year ago. Her physician believes she has acute kidney injury (AKI). A urinalysis does not reveal red or white blood cells (RBCs, WBCs) or cellular casts. Which is the most likely diagnosis in this case?
Answer
  • A. Prerenal AKI.
  • B. Hemodynamically mediated AKI.
  • C. Intrinsic AKI.
  • D. Postrenal AKI

Question 5

Question
R.Z. is a 45-year-old man who presents to the hospital with a rash and “feeling ill.” Physical examination reveals a slightly distended abdomen. His basic metabolic panel reveals sodium (Na) of 140 mEq/L; chloride (Cl) of 108 mEq/L; potassium (K) of 4.5 mEq/L; CO2 of 29 mEq/L; BUN (blood urea nitrogen) of 30 mg/dL; serum creatinine (SCr) of 2.0 mg/dL; and glucose of 148 mg/dL. He recently was initiated on an unknown oral antibiotic for management of an upper respiratory infection. An SCr 6 months ago was 0.9 mg/dL. He has no other medical problems. Which one of the following is the most likely diagnosis in this patient?
Answer
  • A. Acute tubular necrosis.
  • B. Functional acute kidney failure.
  • C. Acute allergic interstitial nephritis.
  • D. Post-renal AKI (acute kidney injury).

Question 6

Question
If a patient with chronic renal failure is treated with erythropoietin (EPO), which of the following will be expected in this patient?
Answer
  • A. Decreased pure red cell aplasia
  • B. Decreased risk of hypertension
  • C. Decreased risk of thrombosis
  • D. Increased well being
  • E. Reduced appetite

Question 7

Question
A 45-year-old lady presents with fatigue and lethargy, and has established end stage renal failure. She has been on haemodialysis for the past three years and receives dialysis for three hours three times a week at a regional haemodialysis centre. At one of her regular visits for haemodialysis she is found to have the following observations and investigations. Blood Pressure 170/95 Serum K+ 5.7 mmol/L (3.5-4.9) Serum corrected calcium 2.0 mmol/L (2.2-2.6) Hb 9.0 g/dL (11.5-16.5) Creatinine 1300 mol/L (60-110) Post-dialysis her blood pressure is recorded as 160/95 mmHg. Which of the following is the best option for managament of this lady's symptoms?
Answer
  • A. Increase dialysis hours
  • B. Treat anaemia with erythropoietin
  • C. Treat hyperkalaemia
  • D. Treat hypocalcaemia

Question 8

Question
A 40-year-old man presents with acute weakness and palpitations. Investigations reveal: Sodium 143 mmol/L (137-144) Potassium 8.0 mmol/L (3.5-4.9) Urea 35 mmol/L (2.5-7.5) Creatinine 450 μmol/L (60-110) Bicarbonate 5 mmol/L (20-28) What is the best immediate therapy?
Answer
  • A. Intravenous calcium gluconate
  • B. Intravenous dextrose and insulin
  • C. Intravenous sodium bicarbonate
  • D. Nebulised salbutamol

Question 9

Question
A 30-year-old male becomes anuric following abdominal surgery. Investigations reveal: Serum urea 9.2 mmol/L (2.5-7.5) Serum creatinine 120 mol/L (60-110) The blood pressure is 130/70 mmHg. What is the next single most appropriate course of action in the management of the anuria?
Answer
  • A. 1 L normal saline intravenously over one hour
  • B. Commence intravenous low-dose dopamine
  • C. Give an 80 mg intravenous bolus of furosemide
  • D. Urgent renal ultrasound

Question 10

Question
Which of the following secondary complications can usually be effectively treated with chronic dialysis therapy (either peritoneal dialysis or hemodialysis) alone?
Answer
  • A) Iron deficiency anemia
  • B) Metabolic acidosis
  • C) Renal osteodystrophy
  • D) Hyperlipidemia

Question 11

Question
Which of the following diuretic regimens would likely be required to optimize diuresis in an individual with stage 4 CKD (GFR 20 mL/min/1.73 m2 [0.19 mL/s/m2]) requiring volume removal?
Answer
  • A) Metolazone alone
  • B) Hydrochlorothiazide + metolazone
  • C) Furosemide + metolazone
  • D) Furosemide + spironolactone

Question 12

Question
Which of the following iron preparations requires a test dose because of the association with anaphylactic reactions?
Answer
  • A) Iron dextran
  • B) Sodium ferric gluconate
  • C) Iron sucrose
  • D) Ferumoxytol

Question 13

Question
A 65-year-old Asian man presents with a history of congestive heart disease, hypertension, osteoarthritis, gastric ulcer, and dyslipidemia. He is admitted to the hospital for a urinary tract infection caused by gram-negative bacteria that is sensitive to trimethoprim/sulfamethoxazole. The patient’s current medication list includes the following: acetaminophen 650 mg orally every 6 hours, carvedilol 6.25 mg orally twice daily, cimetidine 400 mg orally twice daily, digoxin 0.125 mg orally daily, furosemide 40 mg orally daily, gemfibrozil 600 mg orally twice daily 30 minutes before breakfast and dinner, lisinopril 20 mg orally daily, and trimethoprim/sulfamethoxazole 800 mg/240 mg intravenously every 12 hours. Which best reflects the drug combination that causes false elevation of serum creatinine by competing with creatinine for renal secretion?
Answer
  • A. Cimetidine and trimethoprim/sulfamethoxazole.
  • B. Gemfibrozil and digoxin.
  • C. Furosemide and carvedilol.
  • D. Acetaminophen and lisinopril.

Question 14

Question
Hemodynamically mediated kidney injury induced by angiotensin-converting enzyme inhibitors involves all of the following except
Answer
  • A) Enhanced efferent arteriolar constriction
  • B) Patients with renal artery stenosis at increased risk
  • C) Decrease in glomerular capillary hydrostatic pressure
  • D) Reduced glomerular ultrafiltration

Question 15

Question
In addition to lowering PTH, the calcimimetic agent cinacalcet causes which of the following changes in laboratory parameters?
Answer
  • A) Increase in phosphorus, decrease in calcium
  • B) Derease in phosphorus, increase in calcium
  • C) Increase in phosphorus and calcium
  • D) Decrease in phosphorus and calcium

Question 16

Question
A 66-year-old woman with a history of diabetes and chronic kidney disease is scheduled for diagnostic imaging requiring contrast dye administration. Her serum creatinine is 2 mg/dL (177 μmol/L), blood urea nitrogen (BUN) is 30 mg/dL (10.7 mmoL/L), and urine output in the last 24 hours is 1,500 mL. Her complete blood count and electrolytes are all within normal range. Which of the following medications would you recommend to decrease the risk of contrast-induced nephropathy in this patient?
Answer
  • A) Tight glycemic control
  • B) Sodium bicarbonate infusion
  • C) Hemodialysis
  • D) Furosemide infusion
  • E) Low dose dopamine

Question 17

Question
18- The preferred treatment for a patient with drug-induced minimal change glomerular injury accompanied by interstitial nephritis is
Answer
  • A) Amifostine
  • B) Cyclophosphamide
  • C) Pamidronate
  • D) Prednisone
  • E) Hydration

Question 18

Question
Which of the following phosphate-binding agents would be recommended for an individual who has difficulty swallowing larger pills?
Answer
  • A) Lanthanum carbonate tablets
  • B) Aluminum hydroxide liquid
  • C) Calcium carbonate tablets
  • D) Sevelamer carbonate tablets

Question 19

Question
ES is a 45-year-old male on HD started on an epoetin dose of 5,000 Units intravenously TIW 1 week ago. The Hb at the time of initial dosing was 10 g/dL (100 g/L; 6.21 mmol/L). The current Hb is 10.3 g/dL (103 g/L; 6.39 mmol/L). Iron indices reveal the following: ferritin 250 ng/mL transferrin saturation 30% Which of the following options is most appropriate for ES?
Answer
  • A) Oral ferrous Sulfate 325 mg three times per day
  • B) 1g Iron sucrose divided on 10 hemodialysis sessions
  • c) Increase the dose of Epoetin alfa by 25% to achieve the target Hb
  • D) No change is necessary.

Question 20

Question
T.T. is a 50-year-old man who has a long history of type 2 diabetes mellitus, hypertension, and hyperlipidemia. He has had poor medical follow-up and has been maintained on just hydrochlorothiazide 12.5 mg/day for the past several years. At a visit 2 weeks ago, his blood pressure (BP) was 151/92 mm Hg, so he was initiated on enalapril 5 mg/day. His serum creatinine (SCr) at that visit was 1.1 mg/dL, and a serum potassium was 4.3 mEq/L. He returns to the clinic today for a follow-up visit. His BP is 135/75 mm Hg. A serum potassium is 4.5 mEq/L. His SCr is elevated 1.7 mg/dL. He is without complaints. Which one of the following is the most appropriate recommendation to make in this patient?
Answer
  • A. Continue current regimen.
  • B. Discontinue enalapril; start diltiazem.
  • C. Reduce enalapril dose.
  • D. Discontinue hydrochlorothiazide.
Show full summary Hide full summary

Similar

Translations and transformations of functions
Christine Laurich
Waves
kate.siena
Computer science quiz
Ryan Barton
CCNA Security 210-260 IINS - Exam 1
Mike M
CCNA Security 210-260 IINS - Exam 2
Mike M
AQA Biology B1 Questions
Bella Statham
AQA Biology B2 Questions
Bella Statham
AQA Physics P1 Quiz
Bella Statham
GCSE AQA Biology 1 Quiz
Lilac Potato
AQA GCSE Product Design Questions
Bella Statham
General Knowledge Quiz
Andrea Leyden